2
$\begingroup$

Let, \begin{align} f(x,y)=\frac{1}{y^2}+\left(\frac{x}{y}+\sqrt{2-\frac{1-x^2}{y^2}}\right)^2, \end{align}

where $0 \le x\le 1$, $-1 \le y \le0$, $x^2 + y^2 \le 1$ and $x^2 + 2y^2 \ge 1$. What is the $\min\limits_{x,y}f(x,y)$ subject to these constraints?


$f(x,y)$ is not convex under these constraints because $0 \le x\le 1$, $-1 \le y \le0$, $x^2 + y^2 \le 1$ and $x^2 + 2y^2 \ge 1$ is not a convex set.

$\endgroup$

2 Answers 2

2
$\begingroup$

Given, \begin{align} f(x,y)=\frac{1}{y^2}+\left(\frac{x}{y}+\sqrt{2-\frac{1-x^2}{y^2}}\right)^2, \end{align}

where $0 \le x\le 1$, $-1 \le y \le0$, $x^2 + y^2 \le 1$ and $x^2 + 2y^2 \ge 1$.

We can find the $\min\limits_{x,y}f(x,y)$ when $\nabla f(x,y)=0$

Thus, we can expand $f(x,y)$ to become:

$$f(x,y) = 2 + \frac{2 x^2}{y^2} + \frac{2 x}{y}\sqrt{2 - \frac{1}{y^2} + \frac{x^2}{y^2}}$$

Then, the gradient becomes:

$$\nabla f(x,y) = \left( \frac{4xy\sqrt{\frac{x^2+2y^2-1}{y^2}}+4x^2+4y^2-2}{y^3\sqrt{\frac{x^2+2y^2-1}{y^2}}}, \frac{4x^2y\sqrt{\frac{x^2+2y^2-1}{y^2}}+4x^3+4xy^2-4x}{y^4\sqrt{\frac{x^2+2y^2-1}{y^2}}}\right)$$

The minimum of the function will happen when $\nabla f(x,y) = 0$, thus we must solve for such a situation:

The first part:

$$\frac{4xy\sqrt{\frac{x^2+2y^2-1}{y^2}}+4x^2+4y^2-2}{y^3\sqrt{\frac{x^2+2y^2-1}{y^2}}}=0,$$ $$2xy\sqrt{\frac{x^2+2y^2-1}{y^2}}+2x^2+2y^2-1=0,$$ $$2x^2y\sqrt{\frac{x^2+2y^2-1}{y^2}}+2x^3+2xy^2-x=0$$


The second part:

$$\frac{4x^2y\sqrt{\frac{x^2+2y^2-1}{y^2}}+4x^3+4xy^2-4x}{y^4\sqrt{\frac{x^2+2y^2-1}{y^2}}} = 0,$$

$$x^2y\sqrt{\frac{x^2+2y^2-1}{y^2}}+x^3+xy^2-x = 0,$$ $$x^2y\sqrt{\frac{x^2+2y^2-1}{y^2}}=-x^3-xy^2+x$$


We can substitute the second part into the first part like so:

$$2(-x^3-xy^2+x)+2x^3+2xy^2-x=0$$ $$x=0$$

Thus, the minimum will occur when $x=0$. Thus, we plug in $x=0$ to the original function like so:

$$f(y)=\frac{1}{y^2}+2-\frac{1}{y^2}$$

where $x=0$, $-1 \le y \le0$, $y^2 \le 1$ and $2y^2 \ge 1$.

Which then reduces to:

$$f(y)=2$$

where $x=0$, $-1 \le y \le0$, $y^2 \le 1$ and $2y^2 \ge 1$.

However, the constraint $y^2 \le 1$ is equivalent to $-1\le y \le 1$, but is dominated by $-1 \le y \le 0$, and $2y^2 \ge 1$ is equivalent to $y^2 \ge \frac{1}{2}$ which is equivalent to $y \ge \frac{1}{\sqrt{2}}$ or $y \le -\frac{1}{\sqrt{2}}$.

Thus, the minimum will occur when $x=0$, and for any value of $y$ that satisfies $-1\le y \le -\frac{1}{\sqrt{2}}$.

$\endgroup$
7
  • $\begingroup$ Thanks, let me check everything! $\endgroup$ Commented May 2, 2023 at 16:16
  • $\begingroup$ Thanks, so the idea is to forget about the constraints and calculate the minimum of $f(x,y)$ and then see if $\argmin f(x,y)$ falls in the region determined by the constraints which is the case here. Right? $\endgroup$ Commented May 2, 2023 at 18:21
  • $\begingroup$ Correct! As it's easier to find a part of the local solution (since it's non-convex) and then try to find nearby points that fit the other points that local solution $\endgroup$
    – JJMae
    Commented May 2, 2023 at 18:32
  • $\begingroup$ Could you please be more clear? I do not follow what you try to say here! $\endgroup$ Commented May 2, 2023 at 18:35
  • 1
    $\begingroup$ Thanks, that make sense! $\endgroup$ Commented May 2, 2023 at 19:04
1
$\begingroup$

I'd plot this in Desmos, see below:

https://www.desmos.com/calculator/d8fof2su7r

as you can see, the minimum lies tangent to the curve $x^2+y^2=1$. To find this point exactly, try to compute for which $x$ and $y$ these curves have equal gradient. Or substitute $x^2 = 1-y^2$ into $f$. This would give you a function with one variable, which you should be able to investigate in more detail.

$\endgroup$
1
  • $\begingroup$ Thanks, I am interested when $a=1$. In this the above analysis is not valid anymore! Am I right here? $\endgroup$ Commented May 2, 2023 at 15:33

You must log in to answer this question.

Not the answer you're looking for? Browse other questions tagged .